Finding Coordinate Matrix for Linear Transformation T

Click For Summary
The discussion centers on finding the coordinate matrix for a linear transformation T defined on the polynomial space P2, using the ordered basis F = (f1, f2, f3). The user initially attempts to calculate T(f1(t)) but only partially succeeds, obtaining correct coefficients for some terms while struggling with others. They express confusion about how to apply the transformation to the other basis functions, f2(t) and f3(t), and how these relate to the standard basis. Clarification is sought on the correct approach to derive the full coordinate matrix [T]FF based on the transformation's output relative to the specified basis. Understanding the transformation's application to each basis function is crucial for completing the matrix representation accurately.
Rifscape
Messages
41
Reaction score
0

Homework Statement


Hey, I posted another question yesterday, and thanks to the kindness and brilliance of hall of ivy, I was able to solve it. However when I apply the same logic to this new question I cannot seem to get it, can someone explain or show me how to do this question.

Consider the linear transformation T from
V = P2
to
W = P2
given by
T(a0 + a1t + a2t^2) = (2a0 + 4a1 + 2a2) + (2a0 + 3a1 + 4a2)t + (−2a0 + 3a1 + 4a2)t^2
Let F = (f1, f2, f3) be the ordered basis in P2 given by f1(t) = 1, f2(t) = 1 + t, f3(t) = 1 + t + t^2
Find the coordinate matrix [T]FF
of T relative to the ordered basis F used in both V and W, that is, fill in the blanks below: (Any entry that is a fraction should be entered as a proper fraction, i.e. as either x/y or -x/y where x and y are positive integers with no factors in common.)
T(f1(t)) = _f1(t) + _f2(t) + _f3(t)
T(f2(t)) = _f1(t) + _f2(t) + _f3(t)
T(f3(t)) = f1(t) + f2(t) + f3(t)
and therefore :
[T]FF
= the matrix created by the coefficients of the answers above

Homework Equations


Basis equation.

The Attempt at a Solution


I tried doing what I did for the previous question. Since f1(t) = 1. It is 1 + 0t + 0t^2 = 1. So when I plugged in this to the T(a0 + a1t + a2t^2) I got 2 + 2t - 2t^2 for the coefficients of the first matrix. However when I enter this answer in, it is wrong except for the -2 coefficient for t^2. I have no idea why. I think it has something to do with how f2(t) is 1 + t, and how f2(t) is 1 + t + t^2. Could someone please help me on this?

Thank you for your time.
 
Physics news on Phys.org
Rifscape said:

Homework Statement


Hey, I posted another question yesterday, and thanks to the kindness and brilliance of hall of ivy, I was able to solve it. However when I apply the same logic to this new question I cannot seem to get it, can someone explain or show me how to do this question.

Consider the linear transformation T from
V = P2
to
W = P2
given by
T(a0 + a1t + a2t^2) = (2a0 + 4a1 + 2a2) + (2a0 + 3a1 + 4a2)t + (−2a0 + 3a1 + 4a2)t^2
Let F = (f1, f2, f3) be the ordered basis in P2 given by f1(t) = 1, f2(t) = 1 + t, f3(t) = 1 + t + t^2
Find the coordinate matrix [T]FF
of T relative to the ordered basis F used in both V and W, that is, fill in the blanks below: (Any entry that is a fraction should be entered as a proper fraction, i.e. as either x/y or -x/y where x and y are positive integers with no factors in common.)
T(f1(t)) = _f1(t) + _f2(t) + _f3(t)
T(f2(t)) = _f1(t) + _f2(t) + _f3(t)
T(f3(t)) = f1(t) + f2(t) + f3(t)
and therefore :
[T]FF
= the matrix created by the coefficients of the answers above

Homework Equations


Basis equation.

The Attempt at a Solution


I tried doing what I did for the previous question. Since f1(t) = 1. It is 1 + 0t + 0t^2 = 1. So when I plugged in this to the T(a0 + a1t + a2t^2) I got 2 + 2t - 2t^2 for the coefficients of the first matrix.
This is not the "first matrix." This is ##T(f_1)##, the first column of a matrix representation of T. Now calculate ##T(f_2)## and ##T(f_3)##, which will be, respectively, the 2nd and 3rd columns of your matrix.

This matrix won't be the answer, as those three column vectors are in terms of the standard basis ({1, t, t2}), not the basis of this problem.
Rifscape said:
However when I enter this answer in, it is wrong except for the -2 coefficient for t^2. I have no idea why. I think it has something to do with how f2(t) is 1 + t, and how f2(t) is 1 + t + t^2. Could someone please help me on this?

Thank you for your time.
 
Question: A clock's minute hand has length 4 and its hour hand has length 3. What is the distance between the tips at the moment when it is increasing most rapidly?(Putnam Exam Question) Answer: Making assumption that both the hands moves at constant angular velocities, the answer is ## \sqrt{7} .## But don't you think this assumption is somewhat doubtful and wrong?

Similar threads

  • · Replies 7 ·
Replies
7
Views
2K
  • · Replies 2 ·
Replies
2
Views
1K
  • · Replies 18 ·
Replies
18
Views
2K
  • · Replies 2 ·
Replies
2
Views
2K
  • · Replies 4 ·
Replies
4
Views
2K
  • · Replies 3 ·
Replies
3
Views
2K
Replies
8
Views
2K
Replies
1
Views
1K
  • · Replies 1 ·
Replies
1
Views
1K
  • · Replies 11 ·
Replies
11
Views
2K